¿Por qué la divergencia del potencial vectorial puede ser cualquier cosa?

Purcell en su libro estaba derivando el vector potencial A usando rizo ( rizo A ) = m 0 j .

Después de un poco de álgebra, llegó a esto:

2 A X X 2 2 A X y 2 2 A X z 2 + X ( A X X + A y y + A z z ) = m 0 j X .

Luego escribió:

[...] La cantidad entre paréntesis es la divergencia de A . Ahora tenemos cierta libertad en la construcción de A . Todo lo que nos importa es su rizo; su divergencia puede ser cualquier cosa que queramos. Exijamos que

división A = 0
....

Dio la razón:

Para ver por qué somos libres de hacer esto, supongamos que tuviéramos una A tal que rizo A = B , pero división A = F ( X , y , z ) 0. Tratando F como la densidad de carga ρ en el campo electrostático, obviamente encontramos un campo F , el análogo de mi , tal que división F = F . Pero sabemos que el rotacional de tal campo es cero. Por lo tanto, podríamos agregar F a A , creando un nuevo campo con el rotacional correcto y divergencia cero.

Tengo problemas para entender su razón.

Primero, no debería F ser igual a A en sí mismo como la divergencia de ambos es F ? En segundo lugar, ¿por qué debería agregar F a A - ¿No anularía la función ya que ambos F & A son iguales no?

¿Puede alguien por favor ayudarme a entender su razonamiento de por qué somos libres de tomar cualquier cosa por división A ?

Configuración A = 0 se conoce como la medida de Coulomb.

Respuestas (4)

Primero, lo físico que nos importa es B . Así que podemos hacer cualquier cosa para A nos gusta mientras consigamos lo mismo B . Es decir, podemos hacer cualquier cosa que no cambie el rizo de A .

Ahora, supongamos que A = F . Aquí es donde Purcell se olvida de enfatizar lo que quiere decir con "análogo de mi en electrostática" - ¡la curvatura de un campo electrostático es cero ! Entonces F es un campo con F = F , pero × F = 0 . Entonces A F ahora tiene cero divergencia, pero sigue siendo el mismo rotacional.

Aunque la divergencia de A & F es F , no son iguales, ya que el primero tiene una curvatura distinta de cero, mientras que el último tiene una curvatura cero. ¿Tengo razón?
@user36790: Sí.

Debe buscar el teorema de Helmholtz y resultados similares que básicamente le darán la respuesta de ACuriousMind .

Pero una forma en que me gusta visualizar esto es a través de la transformada de Fourier; en el espacio de Fourier el rizo X × X y divergencia X X conviértete simplemente en la cruz X ~ k × X ~ y escalar X ~ k X ~ producto con el vector de onda k . Así que miramos nuestro campo vectorial transformado X ~ en el espacio de Fourier: en cualquier punto PAG , el componente X ~ de X ~ a lo largo del rayo que une el origen y PAG es la parte que contribuye a la divergencia de X , y solo esta parte puede contribuir a la divergencia. Asimismo, la componente en el plano normal a k es el componente que contribuye al rizo, y solo esta parte contribuye al rizo.

Lo que dice Purcell es que somos libres de elegir el componente de X ~ a lo largo del vector de onda para ser lo que queramos.

Entonces, dada una divergencia arbitraria (aparte de las condiciones habituales de convergencia) X , podemos encontrar el componente requerido ϕ ~ ( k ) en el espacio de Fourier resolviendo la ecuación de Poisson 2 ϕ = X .

El término de búsqueda que desea es "transformación de calibre", y si lee sobre eso (google ofrece muchos buenos resultados), encontrará muchas formas diferentes de pensar sobre este problema.

Pero parece que lo que te tiene confundido es esto: creo que estás asumiendo que si dos funciones tienen la misma divergencia, entonces deben ser iguales. Pero piensa en lo que eso implicaría si fuera cierto:

Si A = B implícito A = B , luego definiendo C = A B nos permitiría demostrar que C = 0 implica C = 0 . Pero esto no puede ser correcto, ya que sabemos que mi = 0 gobierna el campo eléctrico en el espacio vacío, y hay muchos campos eléctricos distintos de cero permitidos en el espacio vacío.

Intuitivamente, el gradiente es un campo escalar con menos grados de libertad que el campo vectorial. Por lo tanto, especificar el gradiente no debería brindarle suficiente información, en general, para determinar el campo. Necesita más ecuaciones de restricción que eso.

Entendamos esto conceptualmente. Tenemos B = 0 , por lo tanto requerimos una función vectorial A tal que B = × A de modo que su divergencia se desvanece. Como × B = m 0 j , podemos escribir ( A ) 2 A = m 0 j . Ahora, es posible elegir A tal que su curvatura permanece inalterada pero su divergencia se desvanece. Nos interesa este caso particular porque entonces obtendríamos 2 A = m 0 j que se parece a la ecuación de Poisson para el potencial escalar en electrostática.

Hola, he formateado tu publicación usando MathJax , que es el estándar para este sitio.